On a problem in additive number theory (Q5920248)

From MaRDI portal
Revision as of 12:08, 12 December 2023 by Importer (talk | contribs) (‎Created a new Item)
(diff) ← Older revision | Latest revision (diff) | Newer revision → (diff)
scientific article; zbMATH DE number 6028270
Language Label Description Also known as
English
On a problem in additive number theory
scientific article; zbMATH DE number 6028270

    Statements

    On a problem in additive number theory (English)
    0 references
    0 references
    0 references
    26 April 2012
    0 references
    Let \({\mathbb N}=\{0,1,2,\dots\}\) be the set of all natural numbers. For a sequence of integers \(A=\{a_1<a_2<\cdots\}\), let \(P(A)=\{\sum\varepsilon_i a_i\mid\varepsilon_i=0,1,\;\sum\varepsilon_i<\infty\}\). The authors prove that if \(B=\{b_1<b_2<\cdots\}\) is a sequence of integers with \(b_1\in\{4,7,8\}\cup\{b\,| \,b\geq 11, b\in {\mathbb N}\}\) and \(b_{n+1}\geq 3 b_n+5\) for all \(n\geq 1\), then there exists a sequence of positive integers \(A=\{a_1<a_2<\cdots\}\) for which \(P(A)= {\mathbb N}\backslash B\). On the other hand, if \(B=\{b_1<b_2<\cdots\}\) is a sequence of positive integers with \(b_1\in\{3,5,6,9,10\}\) or \(b_2=3b_1+4\) or \(b_1=1\), \(b_2=9\) or \(b_1=2\), \(b_2=15\), then there is no sequence of positive integers \(A=\{a_1<a_2<\cdots\}\) for which \(P(A)= {\mathbb N}\backslash B\).
    0 references
    0 references
    subset sum
    0 references
    representation problem
    0 references
    Burr's problem
    0 references
    completement
    0 references